Тензорное произведение двух частиц со спином 1

Я довольно смущен, и я надеялся, что кто-нибудь может помочь мне разобраться в этой (вероятно, довольно элементарной) проблеме. У меня есть две частицы со спином 1, состояние которых я описываю как м С и м я соответственно. Оба могут принимать значения -1, 0 и 1.

Теперь я хочу вычислить тензорное произведение. Здесь я, возможно, уже ошибаюсь, но мне, конечно, нужно выбрать основу. Можно ли было бы сказать, что | м с "=" 1 "=" ( 1 0 0 ) , | м с "=" 0 "=" ( 0 1 0 ) , | м с "=" 1 "=" ( 0 0 1 ) , | м я "=" 1 "=" ( 1 0 0 ) , | м я "=" 0 "=" ( 0 1 0 ) , | м я "=" 1 "=" ( 0 0 1 )

Я полагаю, что это может быть уже то, где все начинает идти не так, как может быть, я не могу выбрать одни и те же базисные векторы для двух частиц? В любом случае, ЕСЛИ вышеизложенное верно, я бы, например, хотел рассчитать

1 2 ( | м с "=" 0 я | м с "=" 1 ) | м я "=" 1

Теперь, как я это вижу, это будет просто вектор 9x1, равный

( 0 0 0 0 0 1 2 0 0 я 2 )

Однако я не уверен, что то, что я пишу здесь, правильно. В следующей части моих расчетов я ввожу гамильтоновский член, который заставляет спин первой частицы эволюционировать в зависимости от спина второй частицы, и при вычислении членов кажется, что все идет ужасно неправильно. Поскольку гамильтониан представляет собой просто постоянное произведение тензорного произведения двух z-паули-матриц спина для частицы со спином 1, там не так уж много вещей, которые могут пойти не так, поэтому я решил, что ошибка должна быть где-то здесь.

Хм, быстрый комментарий, я только что заметил, что сделал что-то не так в той части, где не так много могло пойти не так. Может пост удалю.
Еще лучше: опубликуйте решение.

Ответы (1)

Две частицы м с и м я живут в разных векторных пространствах, поэтому на самом деле вы не выбираете одни и те же базисные векторы (поскольку базисные векторы разных частиц принадлежат двум отдельным векторным пространствам).

Во-вторых, тензорное произведение базисных векторов двух разных векторных пространств сформирует базисные векторы нового 3 × 3 "=" 9 размерное векторное пространство. Например:

| м с "=" 1 | м я "=" 0 "=" ( 0 1 0 0 0 0 0 0 0 )
Значит, уравнение, которое вы записали, верно.

Я настоятельно рекомендую эти краткие заметки , чтобы лучше понять весь механизм.

Изменить в ответ на комментарий:

Нам нужно использовать тензорное произведение, чтобы добавить угловые моменты. Для простоты рассмотрим систему с двумя частицами, спин обеих частиц равен 1 / 2 . Затем эти частицы можно объединить следующим образом:

↑↑ , ↑↓ , ↓↑ , ↓↓
Это означает, что двухчастичное гильбертово пространство (т. е. гильбертово пространство, соответствующее системе) натянуто на четыре базисных вектора:
| с 1 , м 1 ; с 2 , м 2 | с 1 , м 1 | с 2 , м 2
Впоследствии, как я набросал в этом ответе , мы разложим это тензорное произведение, чтобы определить, каковы возможные собственные значения для величины и г -компонента полной системы (т.е. система, образованная двумя спин- 1 / 2 частицы) может быть.

Я уверен, что что-то упускаю, но разве мы не добавляем собственные состояния углового момента? Так не должен ли ответ быть линейной комбинацией состояний, взвешенных по их коэффициентам Клебша-Гордана?
@JeffDror Я обновил свое сообщение. Надеюсь, это ответит на ваш вопрос (примерно).
@JeffDror также, ОП напрямую не просит добавить угловой момент двух частиц.
Большое спасибо за помощь, а также особенно за те заметки, очень полезные!
@ user129412 нет проблем. Тензорное произведение всегда казалось мне немного волшебным, пока я не прочитал эти заметки. Поэтому я рад слышать, что вы нашли их полезными.